Приведите формулу определения напряженности электрического поля: Формула напряженности электрического поля в физике

Формула напряженности электрического поля в физике

Содержание:

Определение и формула напряженности электрического поля

Определение

Вектор напряженности $\bar{E}$ – это силовая характеристика электрического поля. В некоторой точке поля, напряженность равна силе, с которой поле действует на единичный положительный заряд, размещенный в указанной точке, при этом направление силы и напряженности совпадают. Математическое определение напряженности записывается так:

$$\bar{E}=\frac{\bar{F}}{q}$$

где $\bar{F}$ – сила, с которой электрическое поле действует на неподвижный, «пробный», точечный заряд q, который размещают в рассматриваемой точке поля. При этом считают, что «пробный» заряд мал на столько, что не искажает исследуемого поля.

Если поле является электростатическим, то его напряженность от времени не зависит.

Если электрическое поле является однородным, то его напряженность во всех точках поля одинакова.

{n} \bar{E}_{i}(2)$$

Допустим, что поле создается системой точечных зарядов и их распределение непрерывно, тогда результирующая напряженность находится как:

$$\bar{E}=\int d \bar{E}(3)$$

интегрирование в выражении (3) проводят по всей области распределения заряда.

Напряженность поля в диэлектрике

Напряженность поля $\bar{E}$ в диэлектрике равна векторной сумме напряженностей полей, создаваемых свободными зарядами $\bar{E}_0$ и связанными (поляризационными зарядами) $\bar{E}_p$:

$$\bar{E}=\bar{E}_{0}+\bar{E}_{p}(4)$$

В том случае, если вещество, которое окружает свободные заряды однородный и изотропный диэлектрик, то напряженность $\bar{E}$ равна:

$$\bar{E}=\frac{\bar{E}_{0}}{\varepsilon}(5)$$

где $\varepsilon$ – относительная диэлектрическая проницаемость вещества в исследуемой точке поля. Выражение (5) обозначает то, что при заданном распределении зарядов напряженность электростатического поля в однородном изотропном диэлектрике меньше, чем в вакууме в $\varepsilon$ раз.

{-12}$ Ф/м (система СИ) — электрическая постоянная.

Связь напряженности и потенциала

В общем случае напряженность электрического поля связана с потенциалом как:

$$\bar{E}=-\operatorname{grad} \varphi-\frac{\partial \bar{A}}{\partial t}(7)$$

где $\varphi$ – скалярный потенциал, $\bar{a}$ – векторный потенциал.

Для стационарных полей выражение (7) трансформируется в формулу:

$$\bar{E}=-\operatorname{grad} \varphi(8)$$

Единицы измерения напряженности электрического поля

Основной единицей измерения напряженности электрического поля в системе СИ является: [E]=В/м(Н/Кл)

Примеры решения задач

Пример

Задание. Каков модуль вектора напряженности электрического поля $\bar{E}$ в точке, которая определена радиус- вектором $\bar{r}_{2}=7 \bar{i}+3 \bar{j}$ (в метрах), если электрическое поле создает положительный точечный заряд (q=1Кл), который лежит в плоскости XOY и его положение задает радиус вектор $\bar{r}_{1}=\bar{i}-5 \bar{j}$, (в метрах)?

Решение. {\prime}-\bar{r}\right) d V$

Читать дальше: Формула пути.

Формула напряженности электрического поля в физике

Содержание:

Определение и формула напряженности электрического поля

Определение

Вектор напряженности $\bar{E}$ – это силовая характеристика электрического поля. В некоторой точке поля, напряженность равна силе, с которой поле действует на единичный положительный заряд, размещенный в указанной точке, при этом направление силы и напряженности совпадают. Математическое определение напряженности записывается так:

$$\bar{E}=\frac{\bar{F}}{q}$$

где $\bar{F}$ – сила, с которой электрическое поле действует на неподвижный, «пробный», точечный заряд q, который размещают в рассматриваемой точке поля. При этом считают, что «пробный» заряд мал на столько, что не искажает исследуемого поля.

Если поле является электростатическим, то его напряженность от времени не зависит. {n} \bar{E}_{i}(2)$$

Допустим, что поле создается системой точечных зарядов и их распределение непрерывно, тогда результирующая напряженность находится как:

$$\bar{E}=\int d \bar{E}(3)$$

интегрирование в выражении (3) проводят по всей области распределения заряда.

Напряженность поля в диэлектрике

Напряженность поля $\bar{E}$ в диэлектрике равна векторной сумме напряженностей полей, создаваемых свободными зарядами $\bar{E}_0$ и связанными (поляризационными зарядами) $\bar{E}_p$:

$$\bar{E}=\bar{E}_{0}+\bar{E}_{p}(4)$$

В том случае, если вещество, которое окружает свободные заряды однородный и изотропный диэлектрик, то напряженность $\bar{E}$ равна:

$$\bar{E}=\frac{\bar{E}_{0}}{\varepsilon}(5)$$

где $\varepsilon$ – относительная диэлектрическая проницаемость вещества в исследуемой точке поля. Выражение (5) обозначает то, что при заданном распределении зарядов напряженность электростатического поля в однородном изотропном диэлектрике меньше, чем в вакууме в $\varepsilon$ раз.

{-12}$ Ф/м (система СИ) — электрическая постоянная.

Связь напряженности и потенциала

В общем случае напряженность электрического поля связана с потенциалом как:

$$\bar{E}=-\operatorname{grad} \varphi-\frac{\partial \bar{A}}{\partial t}(7)$$

где $\varphi$ – скалярный потенциал, $\bar{a}$ – векторный потенциал.

Для стационарных полей выражение (7) трансформируется в формулу:

$$\bar{E}=-\operatorname{grad} \varphi(8)$$

Единицы измерения напряженности электрического поля

Основной единицей измерения напряженности электрического поля в системе СИ является: [E]=В/м(Н/Кл)

Примеры решения задач

Пример

Задание. Каков модуль вектора напряженности электрического поля $\bar{E}$ в точке, которая определена радиус- вектором $\bar{r}_{2}=7 \bar{i}+3 \bar{j}$ (в метрах), если электрическое поле создает положительный точечный заряд (q=1Кл), который лежит в плоскости XOY и его положение задает радиус вектор $\bar{r}_{1}=\bar{i}-5 \bar{j}$, (в метрах)?

Решение. {\prime}-\bar{r}\right) d V$

Читать дальше: Формула пути.

Формула напряженности электрического поля в физике

Содержание:

Определение и формула напряженности электрического поля

Определение

Вектор напряженности $\bar{E}$ – это силовая характеристика электрического поля. В некоторой точке поля, напряженность равна силе, с которой поле действует на единичный положительный заряд, размещенный в указанной точке, при этом направление силы и напряженности совпадают. Математическое определение напряженности записывается так:

$$\bar{E}=\frac{\bar{F}}{q}$$

где $\bar{F}$ – сила, с которой электрическое поле действует на неподвижный, «пробный», точечный заряд q, который размещают в рассматриваемой точке поля. При этом считают, что «пробный» заряд мал на столько, что не искажает исследуемого поля.

Если поле является электростатическим, то его напряженность от времени не зависит. {n} \bar{E}_{i}(2)$$

Допустим, что поле создается системой точечных зарядов и их распределение непрерывно, тогда результирующая напряженность находится как:

$$\bar{E}=\int d \bar{E}(3)$$

интегрирование в выражении (3) проводят по всей области распределения заряда.

Напряженность поля в диэлектрике

Напряженность поля $\bar{E}$ в диэлектрике равна векторной сумме напряженностей полей, создаваемых свободными зарядами $\bar{E}_0$ и связанными (поляризационными зарядами) $\bar{E}_p$:

$$\bar{E}=\bar{E}_{0}+\bar{E}_{p}(4)$$

В том случае, если вещество, которое окружает свободные заряды однородный и изотропный диэлектрик, то напряженность $\bar{E}$ равна:

$$\bar{E}=\frac{\bar{E}_{0}}{\varepsilon}(5)$$

где $\varepsilon$ – относительная диэлектрическая проницаемость вещества в исследуемой точке поля. Выражение (5) обозначает то, что при заданном распределении зарядов напряженность электростатического поля в однородном изотропном диэлектрике меньше, чем в вакууме в $\varepsilon$ раз. {-12}$ Ф/м (система СИ) — электрическая постоянная.

Связь напряженности и потенциала

В общем случае напряженность электрического поля связана с потенциалом как:

$$\bar{E}=-\operatorname{grad} \varphi-\frac{\partial \bar{A}}{\partial t}(7)$$

где $\varphi$ – скалярный потенциал, $\bar{a}$ – векторный потенциал.

Для стационарных полей выражение (7) трансформируется в формулу:

$$\bar{E}=-\operatorname{grad} \varphi(8)$$

Единицы измерения напряженности электрического поля

Основной единицей измерения напряженности электрического поля в системе СИ является: [E]=В/м(Н/Кл)

Примеры решения задач

Пример

Задание. Каков модуль вектора напряженности электрического поля $\bar{E}$ в точке, которая определена радиус- вектором $\bar{r}_{2}=7 \bar{i}+3 \bar{j}$ (в метрах), если электрическое поле создает положительный точечный заряд (q=1Кл), который лежит в плоскости XOY и его положение задает радиус вектор $\bar{r}_{1}=\bar{i}-5 \bar{j}$, (в метрах)?

Решение. {\prime}-\bar{r}\right) d V$

Читать дальше: Формула пути.

Формула напряженности электрического поля в физике

Содержание:

Определение и формула напряженности электрического поля

Определение

Вектор напряженности $\bar{E}$ – это силовая характеристика электрического поля. В некоторой точке поля, напряженность равна силе, с которой поле действует на единичный положительный заряд, размещенный в указанной точке, при этом направление силы и напряженности совпадают. Математическое определение напряженности записывается так:

$$\bar{E}=\frac{\bar{F}}{q}$$

где $\bar{F}$ – сила, с которой электрическое поле действует на неподвижный, «пробный», точечный заряд q, который размещают в рассматриваемой точке поля. При этом считают, что «пробный» заряд мал на столько, что не искажает исследуемого поля.

Если поле является электростатическим, то его напряженность от времени не зависит. {n} \bar{E}_{i}(2)$$

Допустим, что поле создается системой точечных зарядов и их распределение непрерывно, тогда результирующая напряженность находится как:

$$\bar{E}=\int d \bar{E}(3)$$

интегрирование в выражении (3) проводят по всей области распределения заряда.

Напряженность поля в диэлектрике

Напряженность поля $\bar{E}$ в диэлектрике равна векторной сумме напряженностей полей, создаваемых свободными зарядами $\bar{E}_0$ и связанными (поляризационными зарядами) $\bar{E}_p$:

$$\bar{E}=\bar{E}_{0}+\bar{E}_{p}(4)$$

В том случае, если вещество, которое окружает свободные заряды однородный и изотропный диэлектрик, то напряженность $\bar{E}$ равна:

$$\bar{E}=\frac{\bar{E}_{0}}{\varepsilon}(5)$$

где $\varepsilon$ – относительная диэлектрическая проницаемость вещества в исследуемой точке поля. Выражение (5) обозначает то, что при заданном распределении зарядов напряженность электростатического поля в однородном изотропном диэлектрике меньше, чем в вакууме в $\varepsilon$ раз. {-12}$ Ф/м (система СИ) — электрическая постоянная.

Связь напряженности и потенциала

В общем случае напряженность электрического поля связана с потенциалом как:

$$\bar{E}=-\operatorname{grad} \varphi-\frac{\partial \bar{A}}{\partial t}(7)$$

где $\varphi$ – скалярный потенциал, $\bar{a}$ – векторный потенциал.

Для стационарных полей выражение (7) трансформируется в формулу:

$$\bar{E}=-\operatorname{grad} \varphi(8)$$

Единицы измерения напряженности электрического поля

Основной единицей измерения напряженности электрического поля в системе СИ является: [E]=В/м(Н/Кл)

Примеры решения задач

Пример

Задание. Каков модуль вектора напряженности электрического поля $\bar{E}$ в точке, которая определена радиус- вектором $\bar{r}_{2}=7 \bar{i}+3 \bar{j}$ (в метрах), если электрическое поле создает положительный точечный заряд (q=1Кл), который лежит в плоскости XOY и его положение задает радиус вектор $\bar{r}_{1}=\bar{i}-5 \bar{j}$, (в метрах)?

Решение. {\prime}-\bar{r}\right) d V$

Читать дальше: Формула пути.

описание, определение единицы измерения, стандартная формула

В природе существует много интересных явлений, которые обычные люди до сих пор полностью не понимают. К этой категории можно отнести напряжённость электрического поля. Несмотря на то что характеристики этого явления определяются довольно просто, воспользоваться им можно далеко не всегда. Это направление больше носит теоретический характер, из-за чего учёные делают основной упор на получение выгоды в краткосрочной перспективе.

Краткое описание

Увидеть невооружённым взглядом электрическое поле (ЭП) невозможно: его можно обнаружить в процессе воздействия на заряженные тела. Удивительно, но прямого касания может и не быть, так как должна присутствовать силовая природа. Ведь всем известно, что наэлектризованные волосы будут притягиваться к другим предметам. Многочисленные исследования смогли доказать, что аналогичный принцип действия имеют гравитационные поля. Этот феномен был впервые описан в законе Кулона.

Стандартная формула электрического поля выглядит так: F = d₁ d₂ / 4 π q q₀ r ².

Расшифровка:

  • d₁ и d₂ — параметры разрядов в кулонах.
  • q ₀ — этим символом может обозначаться только электрическая постоянная.
  • q — показатель диэлектрической проницаемости.
  • F — сила взаимодействия разных зарядов (может измеряться в ньютонах).
  • r — расстояние между двумя рассматриваемыми объектами в метрах.

Благодаря формуле напряжённости электростатического поля можно определить тот факт, что чем дальше находиться от центра, тем меньше будет ощущаться его воздействие. Графически его можно изобразить в виде силовых линий. Итоговое их расположение напрямую зависит от геометрических параметров носителя.

На сегодняшний день специалисты научились выделять несколько разновидностей полей:

  1. Специфические неоднородное. Рассматривается поле вокруг шарообразного или же точечного заряда. Все силовые линии расходятся только в том случае, если этот параметр имеет положительное значение.
  2. Однородное поле. Все силовые линии располагаются исключительно параллельно друг другу. Эксперты утверждают, что идеальным является тот вариант, когда заряженные пластины бесконечны.

Индуцированные электрическим зарядом силовые линии относятся к замкнутому типу. Иная ситуация наблюдается только у вихревого поля, сформированного вокруг меняющегося магнитного потока.

Ключевые особенности

ЭП представлено особым видом материи, которая встречается вокруг заряженных элементарных частиц (протоны и электроны). Специалисты не один десяток лет занимаются изучением такого интересного явления. Им удалось доказать, что именно через ЭП передаётся влияние одного неподвижного заряда к другому. Итоговое воздействие происходит в строгом соответствии с известным во всём мире законом Кулона.

Так как в промежутке этого расстояния нет плотных тел, можно утверждать о существовании определённого невидимого поля. А так как оно связано со специфическими явлениями, то его начали называть электрическим. Такие поля существуют вокруг всех предметов, только из-за их невидимости и скомпенсированности взаимодействия друг на друга создаётся впечатление, что они проявляются.

Базовые параметры

Изобразить формулу напряжённости можно при помощи как математических закономерностей, так и графических приёмов. Последние характеристики относятся к векторной категории, имеющей определённое направление. Все эти нюансы крайне важны, так как во время решения практических задач часто приходится оперировать не стандартным модулем величины, а специфической проекцией вектора на заранее выбранную ось.

К основным свойствам ЭП можно отнести следующие факты:

  • Оно может как притягивать, так и отталкивать.
  • Невидимость для невооружённого глаза (итоговое определение осуществляется через поведение пробного электрического заряда).
  • Всегда присутствует вокруг заряженных частиц, чего нельзя сказать о магнитном поле.
  • Имеет векторное направление.
  • Взаимодействует исключительно с ЭП.
  • Отличается свойствами неоднородности и концентрации (напряжённость).

Электрическое поле можно определить при помощи обычного точного заряда. Если он будет направлен в интересующую точку пространства, то можно выяснить — присутствует ли в этом месте ЭП. Такой метод определения считается наиболее простым и понятным. Интенсивность излучаемого ЭП используется как обозначение напряжённости.

Влияющие на один и тот же заряд силы будут отличаться друг от друга по направлению и размеру в разных измеряемых точках.

Стоит отметить, что закон Кулона не адаптирован под современные требования. Для одной точки поля сила F будет прямо пропорциональна величине точечного заряда. На фоне этого эксперты провели множество исследований. Теперь принято считать силовой характеристикой единицы измерения напряжённости «Е». Этот параметр является векторной величиной. Найти напряжённость электрического поля можно в Ньютонах на Кулон.

Отдельно стоит учесть, что если ЭП образуется сразу несколькими зарядами, то общая напряжённость в определённой точке находится как общая геометрическая сумма.

Изучение потенциала

Именно этот параметр считается распространённой характеристикой ЭП. Потенциал выступает в роли накопленной ценной энергии, используемой для перемещения различных зарядов. В итоге потенциал может весь израсходоваться, из-за чего его показатель будет равен нулю.

Процесс накопления происходит в обратном порядке. В качестве яркого примера можно использовать всё тот же заряд, но находящийся вне ЭП. Только когда определённая сила перемещает его внутрь и постепенно двигает там, появляется необходимый потенциал.

Если человек только столкнулся с этой отраслью и хочет в ней разобраться, то ему лучше представить обычную пружину. В спокойном состоянии у неё отсутствует какой-либо потенциал, из-за чего она может расцениваться только как небольшой металлический предмет. Но как только человек начнёт её постепенно сдавливать, будет образовываться потенциал. Если быстро отпустить пружину, то она мгновенно выпрямится и при этом сдвинет со своего пути все посторонние предметы.

Этот пример ярко демонстрирует то, что уровень потенциала всегда будет соответствовать приложенным усилиям на перемещение заряда. В современной науке этот показатель можно измерить в вольтах.

Сферы применения

Стандартные характеристики ЭП обязательно включают в себя два свойства, которые активно применяются человечеством. Они могут образовывать универсальные ионы, а погруженные в определённую жидкость электроды позволяют без каких-либо усилий разделять их по функциям. Эксперты доказали, что универсальной и доступность электрических полей активно используется в различных отраслях:

  • Очистка. В этой отрасли активно используется система качественного разделения разных жидкостей. Эта функция высоко ценится в очистных сооружениях. Ведь та вода, в которой содержится большое количество различного мусора, очень вредна для человека. При этом с такой жидкостью очень сложно что-то сделать, так как далеко не все фильтры могут справиться с проблемой. Именно в такой ситуации на помощь приходят ЭП. Они разделяют воду, за счёт чего отделяются загрязнения. Благодаря этому можно пользоваться быстрым и доступным способом очистки.
  • Медицина. Квалифицированные доктора практически ежедневно используют систему воздействия на поражённые ткани пациента направленными ионами. За счёт этого улучшается регенерация органа, убиваются микробы и очищается рана. К тому же уникальные характеристики и свойства ЭП позволяют им работать с большей частотой. Такой эффект широко востребован в медицине, так как за короткий промежуток времени можно повысить температуру некоторых отдельных частей тела, за счет чего восстанавливается кровоток, а также улучшается общее самочувствие пациента.
  • Химия. Без электрических полей просто невозможна нормальная работа некоторых отраслей промышленности, где нужно разделять разные жидкости. Такая наука активно используется в стандартных лабораторных условиях, но чаще всего её можно встретить в сфере массовой добычи нефти. Большой спрос спровоцирован тем, что природный материал часто содержит загрязняющие частицы, избавиться от которых традиционным способом весьма проблематично. Более экономичным является применение ЭП. Они позволяют быстро разделить нефть, убрав весь ненужный мусор, облегчив дальнейшую обработку.

Конечно, существует множество других вариантов применения формулы напряжённости электрического поля.

К примеру: эксперты могут применять такое явление в качестве беспроводной системы передачи тока к разным приборам. Но в большинстве случае все такие разработки носят экспериментальный и теоретический характер.

Закон Кулона

В этом случае силовая характеристика электрического поля работает для точечного заряда, находящегося на расстоянии определённого радиуса от него. Если же взять этот показатель по стандартному модулю, то в итоге получится кулоновское поле.

Направление вектора напрямую зависит от имеющегося знака заряда. Если он плюсовой, то ЭП будет «передвигаться» по радиусу. В противном случае сам вектор будет направлен в сторону заряда.

Чтобы разобраться в ключевых особенностях закона, можно изучить основные рисунки и диаграммы, где изображены силовые линии. В учебниках основные характеристики ЭП объясняются довольно сложно. Если же для изучения этой темы использовать специализированную литературу, тогда нужно учесть, что при построении рисунков силовых линий их итоговая густота является пропорциональной модулю вектора напряжённости. Это своего рода подсказка от экспертов, которая может помочь во время экзамена или просто для контроля знаний.

Принцип воздействия

Свойства ЭП чаще всего постоянны и однообразны. Для планеты свойственен свой защитный фон, который на живые организмы практически никак не влияет. Незначительные проявления становятся заметными для человека только во время сильной грозы. В такой ситуации может даже казаться, что воздух дрожит от напряжения. Но для большинства людей это не представляет никакой угрозы.

Индустрия технологий не стоит на месте, благодаря чему специалисты изготавливают всё больше различных агрегатов, каждый из которых способен генерировать собственное ЭП. Показатель существенно превышает естественный фон, который составляет 0.5 кВ/м. Конечно, такая особенность не осталась незамеченной со стороны экспертов. В результате многочисленных проб они вывели максимально допустимое напряжение, которое не создаёт ограничений для человека. Его размер составляет 27 кВ/м.

Даже если включить сразу все бытовые устройства, максимальный показатель не будет превышен. Взрослый человек может получить небольшой процент негативного воздействия только при длительном нахождении возле высоковольтных проводов. В такой среде напряжение очень большое, из-за чего долго стоять или же работать на таком участке категорически запрещено. Специалисты, которые вынуждены по служебным обстоятельствам находиться в окружении таких ЭП, должны успевать выполнить все работы максимум за полтора часа.

Внедрение в технику

Современные масштабы ЭП нашли весьма интересное применение в современном мире. Специалистами был разработан способ беспроводной передачи сигнала от основного источника до потребителя, хотя ещё до недавнего времени всё носило экспериментальный и теоретический характер.

На сегодняшний день уже имеется эффективная реализация технологии зарядки смартфонов без использования гибкого кабеля. Конечно, этот вариант пока не позволяет передавать энергию на дальние расстояния, но все функции находятся в стадии совершенствования.

Стоит отметить, что изучением электрического поля занималось уже много людей. Огромный след в истории оставил известный во всём мире сербский изобретатель Николай Тесла. Благодаря приложенным усилиям ему удалось достичь больших успехов, но не в плане энергетической эффективности.

формула, в чем измеряется, как найти.

Заряженное тело постоянно передает часть энергии, преобразуя ее в другое состояние, одной из частей которого является электрическое поле. Напряженность – основная составляющая, которая характеризует электрическую часть электромагнитного излучения. Его значение зависит от силы тока и выступает силовой характеристикой. Именно по этой причине высоковольтные провода размещают на большую высоту, чем проводку для меньшего тока.

Определение понятия и формула расчета

Вектор напряженности (E) — сила, действующая на бесконечно малый ток в рассматриваемой точке. Формула для определения параметра выглядит следующим образом:

Где:

  • F- сила, которая действует на заряд;
  • q –величина заряда.

Заряд, принимающий участие в исследовании, называется пробным. Он должен быть незначительным, чтобы не искажать результаты. При идеальных условиях в роли q выступает позитрон.

Стоит отметить, что величина относительна, ее количественная характеристика и направление зависят от координат и при смещении изменится.

Исходя из закона кулона сила, действующая на тело, равняется произведению потенциалов, деленному на квадрат расстояния между телами.

F=q1*q2/r2

Из этого следует, что напряженность в данной точке пространства прямо пропорциональна потенциалу источника и обратно пропорциональна квадрату расстояния между ними. В общем, символическом случае уравнение записывается следующим образом:

E=q/r2

Исходя из уравнения, единица измерения электрического поля – Вольт на метр. Это же обозначение принято системой СИ. Имея значение параметра, можно вычислить силу, которая будет действовать на тело в исследуемой точке, а зная силу — найти напряженность электрического поля.

По формуле видно, что результат абсолютно не зависит от пробного заряда. Это необычно, так как данный параметр присутствует в первоначальном уравнении. Однако это логично, потому что источником является основной, а не пробный излучатель. В реальных условиях данный параметр имеет влияние на измеряемые характеристики и выдает искажение, что обуславливает использование позитрона для идеальных условий.

Так как напряженность – векторная величина, кроме значения она имеет направление. Вектор направлен от основного источника к исследуемому, или от пробного заряда к основному. Это зависит от полярности. Если знаки одинаковые, то происходит отталкивание, вектор направлен к исследуемой точке. Если точки заряжены разнополярно, то источники притягиваются. В этом случае принято считать, что вектор силы направлен от положительного источника к отрицательному.

Единица измерения

В зависимости от контекста и применения в областях электростатики напряженность электрического поля [E] измеряется в двух единицах.  Это могут быть вольт/метр или ньютон/кулон. Причиной такой путаницы представляется получение ее из разных условий, выведение единицы измерений из применяемых формул. В некоторых случаях одна из размерностей используется намерено для предотвращения применения формул, которые работают только для частных случаев. Понятие присутствует в фундаментальных электродинамических законах, поэтому величина является для термодинамики базовой.

Принцип суперпозиции

Источник может принимать различные формы. Описанные выше формулы помогают найти напряженность электрического поля точечного заряда, но источник может представлять собой и другие формы:

  • несколько независимых материальных точек;
  • распределенную прямую или кривую (статор электромагнита, провод и т. д.).

Для точечного заряда нахождение напряженности выглядит следующим образом: E=k*q/r2, где k=9*109

При воздействии на тело нескольких источников напряженность в точке будет равняться векторной сумме потенциалов. При действии распределенного источника вычисляется действующим интегралом по всей области распределения.

Характеристика может изменяться во времени в связи с изменением зарядов. Значение остается постоянным только для электростатического поля. Она является одной из основных силовых характеристик, поэтому для однородного поля направление вектора и величина q будут одинаковыми в любых координатах.

С точки зрения термодинамики

Напряженность выступает одним из основных и ключевых характеристик в классической электродинамике. Ее значение, а также данные электрического заряда и магнитной индукции представляются основными характеристиками, зная которые можно определить параметры протекания практически всех электродинамических процессов. Она присутствуют и выполняет важную роль в таких фундаментальных понятиях, как формула силы Лоренца и уравнения Максвелла.

Где:

F-сила Лоуренца;

  • q – заряд;
  • B – вектор магнитной индукции;
  • С – скорость света в вакууме;
  • j – плотность магнитного тока;
  • μ0 – магнитная постоянная = 1,25663706*10-6;
  • ε0 – электрическая постоянная, равная 8,85418781762039*10-12

Наряду со значением магнитной индукцией данный параметр является основной характеристикой электромагнитного поля, излучаемого зарядом. Исходя из этого, с точки зрения термодинамики напряженность – значительно более важное значение, чем сила тока или другие показатели.

Данные законы выступают фундаментальными, на них строится вся термодинамика. Следует отметить, что закон Ампера и другие более ранние формулы являются приближенными или описывают частные случаи. Законы Максвелла и Лоренца универсальны.

Практическое значение

Понятие напряженности нашло широкое применение в электротехнике. Оно применяется для расчетов норм сигналов, вычисления устойчивости системы, определения влияния электрического излучения на окружающие источник элементы.

Основной сферой, где понятие нашло широкое применение, является сотовая и спутниковая связь, телевышки и другие электромагнитные излучатели. Знание интенсивности излучения для данных устройств позволяют рассчитать такие параметры, как:

  • дальность действия радиовышки;
  • безопасное расстояние от источника до человека.

Первый параметр крайне важен для тех, кто устанавливает спутниковое телевизионное вещание, а также мобильную связь. Второй дает возможность определить допустимые нормы по излучению, тем самым обезопасив пользователей от вредного влияния электроприборов. Применение данных свойств электромагнитного излучения не ограничивается связью. На этих базовых принципах построена выработка энергии, бытовая техника, отчасти производство механических изделий (например, окрашивание при помощи электромагнитных импульсов). Таким образом, понимание величины является важным и для производственного процесса.

Интересные опыты, позволяющие увидеть картину силовых линий электрического поля: видео

Читайте также:

Электрическое поле – FIZI4KA

Электродинамика – раздел физики, изучающий свойства и взаимодействия электрических зарядов, осуществляемые посредством электромагнитного поля.

Электростатикой называется раздел электродинамики, в котором рассматриваются свойства и взаимодействия неподвижных электрически заряженных тел или частиц.

Электромагнитное взаимодействие – это взаимодействие между электрически заряженными частицами или макротелами.

Точечный заряд – заряженное тело, размер которого мал по сравнению с расстоянием, на котором оценивается его действие.

Электризация тел

Электризация – процесс сообщения телу электрического заряда, т. е. нарушение его электрической нейтральности. Процесс электризации представляет собой перенесение с одного тела на другое электронов или ионов. В результате электризации тело получает возможность участвовать в электромагнитном взаимодействии.

Способы электризации:

  • трением, – например, электризация эбонитовой палочки при трении о мех. При тесном соприкосновении двух тел часть электронов переходит с одного тела на другое; в результате этого на поверхности у одного из тел создается недостаток электронов и тело получает положительный заряд, а у другого – избыток, и тело заряжается отрицательно. Величины зарядов тел одинаковы;
  • через влияние (электростатическая индукция) – тело остается электрически нейтральным, электрические заряды внутри него перераспределяются так, что разные части тела приобретают разные по знаку заряды;
  • при соприкосновении заряженного и незаряженного тела – заряд при этом распределяется между этими телами пропорционально их размерам. Если размеры тел одинаковы, то заряд распределяется между ними поровну;
  • при ударе;
  • под действием излучения – под действием света с поверхности проводника могут вырываться электроны, при этом проводник приобретает положительный заряд.

Взаимодействие зарядов. Два вида зарядов

Электрический заряд – скалярная физическая величина, характеризующая способность тела участвовать в электромагнитных взаимодействиях.

Обозначение – ​\( q \)​, единица измерения в СИ – кулон (Кл).

Существуют два вида электрических зарядов: положительный и отрицательный. Наименьший отрицательный заряд имеет электрон (–1,6·10-19 Кл), наименьший положительный заряд (1,6·10-19 Кл) – протон. Минимальный заряд, который может быть сообщен телу, равен заряду электрона (элементарный заряд). Если тело имеет избыточные (лишние) электроны, то тело заряжено отрицательно, если у тела недостаток электронов, то тело заряжено положительно.

Величина заряда тела будет равна

где ​\( N \)​ — число избыточных или недостающих электронов;
​\( e \)​ — элементарный заряд, равный 1,6·10-19 Кл.

Важно!
Частица может не иметь заряда, но заряд без частицы не существует.

Электрические заряды взаимодействуют:

  • заряды одного знака отталкиваются:

  • заряды противоположных знаков притягиваются:

Прибор для обнаружения электрического заряда называется электроскоп. Основная часть прибора – металлический стержень, на котором закреплены два листочка металлической фольги, помещенные в стеклянный сосуд. При соприкосновении заряженного тела со стержнем электроскопа заряды распределяются между листочками фольги. Так как заряд листочков одинаков по знаку, они отталкиваются.

Для измерения зарядов можно использовать и электрометр. Основные части его – металлический стержень и стрелка, которая может вращаться вокруг горизонтальной оси. Стержень со стрелкой закреплен в пластмассовой втулке и помещен в металлический корпус, закрытый стеклянными крышками. При соприкосновении заряженного тела со стержнем стержень и стрелка получают электрические заряды одного знака. Стрелка поворачивается на некоторый угол.

Закон сохранения электрического заряда

Систему называют замкнутой (электрически изолированной), если в ней не происходит обмена зарядами с окружающей средой.

В любой замкнутой (электрически изолированной) системе сумма электрических зарядов остается постоянной при любых взаимодействиях внутри нее.

Полный электрический заряд ​\( (q) \)​ системы равен алгебраической сумме ее положительных и отрицательных зарядов ​\( (q_1, q_2 … q_N) \)​:

Важно!
В природе не возникают и не исчезают заряды одного знака: положительный и отрицательный заряды могут взаимно нейтрализовать друг друга, если они равны по модулю.

Закон Кулона

Закон Кулона был открыт экспериментально: в опытах с использованием крутильных весов измерялись силы взаимодействия заряженных шаров.

Закон Кулона формулируется так:
сила взаимодействия ​\( F \)​ двух точечных неподвижных электрических зарядов в вакууме прямо пропорциональна их модулям ​\( q_1 \)​ и \( q_2 \) и обратно пропорциональна квадрату расстояния между ними ​\( r \)​:

где ​\( k=\frac{1}{4\pi\varepsilon_0}=9\cdot10^9 \)​ (Н·м2)/Кл2 – коэффициент пропорциональности,
​\( \varepsilon_0=8.{-12} \)​ Кл2/(Н·м2) – электрическая постоянная.

Коэффициент ​\( k \)​ численно равен силе, с которой два точечных заряда величиной 1 Кл каждый взаимодействуют в вакууме на расстоянии 1 м.

Сила Кулона направлена вдоль прямой, соединяющей взаимодействующие заряды. Заряды взаимодействуют друг с другом с силами, равными по величине и противоположными по направлению.

Значение силы Кулона зависит от среды, в которой они находятся. В этом случае формула закона:

где ​\( \varepsilon \)​ – диэлектрическая проницаемость среды.

Закон Кулона применим к взаимодействию

  • неподвижных точечных зарядов;
  • равномерно заряженных тел сферической формы.

В этом случае ​\( r \)​ – расстояние между центрами сферических поверхностей.

Важно!
Если заряженное тело протяженное, то его необходимо разбить на точечные заряды, рассчитать силы их попарного взаимодействия и найти равнодействующую этих сил (принцип суперпозиции).

Действие электрического поля на электрические заряды

Электрическое поле – это особая форма материи, существующая вокруг электрически заряженных тел.

Впервые понятие электрического поля было введено Фарадеем. Он объяснял взаимодействие зарядов следующим образом: каждый заряд создает вокруг себя электрическое поле, которое с некоторой силой действует на другой заряд.

Свойства электрического поля заключаются в том, что оно:

  • материально;
  • создается зарядом;
  • обнаруживается по действию на заряд;
  • непрерывно распределено в пространстве;
  • ослабевает с увеличением расстояния от заряда.

Действие заряженного тела на окружающие тела проявляется в виде сил притяжения и отталкивания, стремящихся поворачивать и перемещать эти тела по отношению к заряженному телу.

Силу, с которой электрическое поле действует на заряд, можно рассчитать по формуле:

где ​\( \vec{E} \)​ – напряженность электрического поля, ​\( q \)​ – заряд.

Решение задач о точечных зарядах и системах, сводящихся к ним, основано на применении законов механики с учетом закона Кулона и вытекающих из него следствий.

Алгоритм решения задач о точечных зарядах и системах, сводящихся к ним:

  • сделать рисунок; указать силы, действующие на точечный заряд, помещенный в электрическое поле;
  • записать для заряда условие равновесия или основное уравнение динамики материальной точки;
  • выразить силы электрического взаимодействия через заряды и поля и подставить эти выражения в исходное уравнение;
  • если при взаимодействии заряженных тел между ними происходит перераспределение зарядов, к составленному уравнению добавить уравнение закона сохранения зарядов;
  • записать математически все вспомогательные условия;
  • решить полученную систему уравнений относительно неизвестной величины;
  • проверить решение

Напряженность электрического поля

Напряженность электрического поля ​\( \vec{E} \)​ – векторная физическая величина, равная отношению силы ​\( F \)​, действующей на пробный точечный заряд, к величине этого заряда ​\( q \)​:

Обозначение – \( \vec{E} \), единица измерения в СИ – Н/Кл или В/м.9 \) (Н·м2)/Кл2,
​\( q_0 \)​ – заряд, создающий поле,
​\( r \)​ – расстояние от заряда, создающего поле, до данной точки.

Напряженность поля точечного заряда в среде вычисляется по формуле:

где ​\( \varepsilon \)​ – диэлектрическая проницаемость среды.

Важно!
Напряженность электрического поля не зависит от величины пробного заряда, она определяется величиной заряда, создающего поле.

Направление вектора напряженности в данной точке совпадает с направлением силы, с которой поле действует на положительный пробный заряд, помещенный в эту точку.

Линией напряженности электрического поля называется линия, касательная к которой в каждой точке направлена вдоль вектора напряженности ​\( \vec{E} \)​.

Линии напряженности электростатического поля начинаются на положительных электрических зарядах и заканчиваются на отрицательных электрических зарядах или уходят в бесконечность от положительного заряда и приходят из бесконечности к отрицательному заряду.

Распределение линий напряженности вокруг положительного и отрицательного точечных зарядов показано на рисунке.

Определяя направление вектора ​\( \vec{E} \)​ в различных точках пространства, можно представить картину распределения линий напряженности электрического поля.

Поле, в котором напряженность одинакова по модулю и направлению в любой точке, называется однородным электрическим полем. Однородным можно считать электрическое поле между двумя разноименно заряженными металлическими пластинами. Линии напряженности в однородном электрическом поле параллельны друг другу.

Принцип суперпозиции электрических полей

Каждый электрический заряд создает в пространстве электрическое поле независимо от наличия других электрических зарядов.

Принцип суперпозиции электрических полей: напряженность электрического поля системы ​\( N \)​ зарядов равна векторной сумме напряженностей полей, создаваемых каждым из них в отдельности:

Электрические поля от разных источников существуют в одной точке пространства и действуют на заряд независимо друг от друга.

Потенциальность электростатического поля

Электрическое поле с напряженностью ​\( \vec{E} \)​ при перемещении заряда ​\( q \)​ совершает работу. Работа ​\( A \)​ электростатического поля вычисляется по формуле:

где ​\( d \)​ – расстояние, на которое перемещается заряд,
​\( \alpha \)​ – угол между векторами напряженности электрического поля и перемещения заряда.

Важно!
Эта формула применима для нахождения работы только в однородном электростатическом поле.

Работа сил электростатического поля при перемещении заряда из одной точки поля в другую не зависит от формы траектории, а определяется только начальным и конечным положением заряда.

Потенциальным называется поле, работа сил которого по перемещению заряда по замкнутой траектории равна нулю.

Важно!
Работа сил электростатического поля при перемещении заряда по любой замкнутой траектории равна нулю. Электростатическое поле является потенциальным.

Работа электростатического поля по перемещению заряда равна изменению потенциальной энергии, взятому с противоположным знаком. В электродинамике энергию принято обозначать буквой ​\( W \)​, так как буквой ​\( E \)​ обозначают напряженность поля:

Потенциальная энергия заряда ​\( q \)​, помещенного в электростатическое поле, пропорциональна величине этого заряда. Потенциальная энергия взаимодействия зарядов вычисляется относительно нулевого уровня (аналогично потенциальной энергии поля силы тяжести). Выбор нулевого уровня потенциальной энергии определяется исходя из соображений удобства при решении задачи.

Потенциал электрического поля. Разность потенциалов

Потенциал – скалярная физическая величина, равная отношению потенциальной энергии электрического заряда в электростатическом поле к величине этого заряда.

Обозначение – ​\( \varphi \)​, единица измерения в СИ – вольт (В).

Потенциал \( \varphi \) является энергетической характеристикой электростатического поля.

Разность потенциалов численно равна работе, которую совершает электрическая сила при перемещении единичного положительного заряда между двумя точками поля:

Обозначение – ​\( \Delta\varphi \)​, единица измерения в СИ – вольт (В).

Иногда разность потенциалов обозначают буквой ​\( U \)​ и называют напряжением.

Важно!
Разность потенциалов \( \Delta\varphi=\varphi_1-\varphi_2 \), а не изменение потенциала \( \Delta\varphi=\varphi_2-\varphi_1 \). Тогда работа электростатического поля равна:

Важно!
Эта формула позволяет вычислить работу электростатических сил в любом поле.

В электростатике часто вычисляют потенциал относительно бесконечно удаленной точки. В этом случае потенциал поля в данной точке равен работе, которую совершают электрические силы при удалении единичного положительного заряда из данной точки в бесконечность.

Потенциал поля точечного заряда ​\( q \)​ в точке, удаленной от него на расстояние ​\( r \)​, вычисляется по формуле:

Для наглядного представления электрического поля используют эквипотенциальные поверхности.

Важно!
Внутри проводящего шара потенциал всех точек внутри шара равен потенциалу поверхности шара и вычисляется по формуле потенциала точечного заряда (​\( r =R \)​, где ​\( R \)​ – радиус шара). Напряженность поля внутри шара равна нулю.

Эквипотенциальной поверхностью, или поверхностью равного потенциала, называется поверхность, во всех точках которой потенциал имеет одинаковое значение.

Свойства эквипотенциальных поверхностей

  • Вектор напряженности перпендикулярен эквипотенциальным поверхностям и направлен в сторону убывания потенциала.
  • Работа по перемещению заряда по эквипотенциальной поверхности равна нулю.

В случае однородного поля эквипотенциальные поверхности представляют собой систему параллельных плоскостей. Для точечного заряда эквипотенциальные поверхности представляют собой концентрические окружности.

Разность потенциалов и напряженность связаны формулой:

Из принципа суперпозиции полей следует принцип суперпозиции потенциалов:

Потенциал результирующего поля равен сумме потенциалов полей отдельных зарядов.

Важно!
Потенциалы складываются алгебраически, а напряженности – по правилу сложения векторов.

Решение задач о точечных зарядах и системах, сводящихся к ним, основано на применении законов сохранения, теоремы об изменении кинетической энергии заряда с учетом работы электростатических сил.

Алгоритм решения таких задач:

  • установить характер и особенности электростатических взаимодействий объектов системы;
  • ввести характеристики (силовые и энергетические) этих взаимодействий, сделать рисунок;
  • записать законы сохранения и движения для объектов;
  • выразить энергию электростатического взаимодействия через заряды, потенциалы, напряженности;
  • составить систему уравнений и решить ее относительно искомой величины;
  • проверить решение.

Проводники в электрическом поле

Проводниками называют вещества, в которых может происходить упорядоченное перемещение электрических зарядов, т. е. протекать электрический ток.

Проводниками являются металлы, водные растворы солей, кислот, ионизованные газы. В проводниках есть свободные электрические заряды. В металлах валентные электроны взаимодействующих друг с другом атомов становятся свободными.

Если металлический проводник поместить в электрическое поле, то под его действием свободные электроны проводника начнут перемещаться в направлении, противоположном направлению напряженности поля. В результате на одной поверхности проводника появится избыточный отрицательный заряд, а на противоположной – избыточный положительный заряд.

Эти заряды создают внутри проводника внутреннее электрическое поле, вектор напряженности которого направлен противоположно вектору напряженности внешнего поля. Под действием внешнего электростатического поля электроны проводимости в металлическом проводнике перераспределяются так, что напряженность результирующего поля в любой точке внутри проводника равна нулю. Электрические заряды расположены на поверхности проводника.

Важно!
Если внутри проводника есть полость, то напряженность в ней будет равна нулю независимо от того, какое поле имеется вне проводника и как заряжен проводник. Внутренняя полость в проводнике экранирована (защищена) от внешних электростатических полей. На этом основана электростатическая защита.

Явление перераспределения зарядов во внешнем электростатическом поле называется электростатической индукцией.

Заряды, разделенные электростатическим полем, взаимно компенсируют друг друга, если проводник удалить из поля. Если такой проводник разрезать, не вынося из поля, то его части будут иметь заряды разных знаков.

Важно!
Во всех точках поверхности проводника вектор напряженности направлен перпендикулярно к его поверхности. Поверхность проводника является эквипотенциальной (потенциалы всех точек поверхности проводника равны).

Диэлектрики в электрическом поле

Диэлектриками называют вещества, не проводящие электрический ток. Диэлектриками являются стекло, фарфор, резина, дистиллированная вода, газы.

В диэлектриках нет свободных зарядов, все заряды связаны. В молекуле диэлектрика суммарный отрицательный заряд электронов равен положительному заряду ядра. Различают полярные и неполярные диэлектрики.

В молекулах полярных диэлектриков ядра и электроны расположены так, что центры масс положительных и отрицательных зарядов не совпадают и находятся на некотором расстоянии друг от друга. То есть молекулы представляют собой диполи независимо от наличия внешнего электрического поля. В отсутствие внешнего электрического поля из-за теплового движения молекул диполи расположены хаотично, поэтому суммарная напряженность поля всех диполей диэлектрика равна нулю.

Если в отсутствие внешнего электрического поля центры масс положительных и отрицательных зарядов в молекуле диэлектрика совпадают, то он называется неполярным. Пример такого диэлектрика – молекула водорода. Если такой диэлектрик поместить во внешнее электрическое поле, то направления векторов сил, действующих на положительные и отрицательные заряды, будут противоположными. В результате молекула деформируется и превращается в диполь. При внесении диэлектрика в электрическое поле происходит его поляризация.

Поляризация диэлектрика – процесс смещения в противоположные стороны разноименных связанных зарядов, входящих в состав атомов и молекул вещества в электрическом поле.

Если диэлектрик неполярный, то в его молекулах происходит смещение положительных и отрицательных зарядов. На поверхности диэлектрика появятся поверхностные связанные заряды. Связанными эти заряды называют потому, что они не могут свободно перемещаться отдельно друг от друга.

Внутри диэлектрика суммарный заряд равен нулю, а на поверхностях заряды не скомпенсированы и создают внутри диэлектрика поле, вектор напряженности которого направлен противоположно вектору напряженности внешнего поля. Это значит, что внутри диэлектрика поле имеет меньшую напряженность, чем в вакууме.

Физическая величина, равная отношению модуля напряженности электрического поля в вакууме к модулю напряженности электрического поля в однородном диэлектрике, называется диэлектрической проницаемостью вещества:

В полярном диэлектрике во внешнем электрическом поле происходит поворот диполей, и они выстраиваются вдоль линий напряженности.

Если внесенный в электрическое поле диэлектрик разрезать, то его части будут электрически нейтральны.

Электрическая емкость. Конденсатор

Электрическая емкость (электроемкость) – скалярная физическая величина, характеризующая способность уединенного проводника удерживать электрический заряд.

Обозначение – ​\( C \)​, единица измерения в СИ – фарад (Ф).

Уединенный проводник – это проводник, удаленный от других проводников и заряженных тел.

Фарад – электроемкость такого уединенного проводника, потенциал которого изменяется на 1 В при сообщении ему заряда 1 Кл:

Формула для вычисления электроемкости:

где ​\( q \)​ – заряд проводника, ​\( \varphi \)​ – его потенциал.

Электроемкость зависит от его линейных размеров и геометрической формы. Электроемкость не зависит от материала проводника и его агрегатного состояния. Электроемкость проводника прямо пропорциональна диэлектрической проницаемости среды, в которой он находится.

Конденсатор – это система из двух проводников, разделенных слоем диэлектрика, толщина которого мала по сравнению с размерами проводников.

Проводники называют обкладками конденсатора. Заряды обкладок конденсатора равны по величине и противоположны по знаку заряда. Электрическое поле сосредоточено между обкладками конденсатора. Конденсаторы используют для накопления электрических зарядов.

Электроемкость конденсатора рассчитывается по формуле:

где ​\( q \)​ – модуль заряда одной из обкладок,
​\( U \)​ – разность потенциалов между обкладками.

Электроемкость конденсатора зависит от линейных размеров и геометрической формы и расстояния между проводниками. Электроемкость конденсатора прямо пропорциональна диэлектрической проницаемости вещества между проводниками.

Плоский конденсатор представляет две параллельные пластины площадью ​\( S \)​, находящиеся на расстоянии ​\( d \)​ друг от друга.

Электроемкость плоского конденсатора:

где ​\( \varepsilon \)​ – диэлектрическая проницаемость вещества между обкладками,
\( \varepsilon_0 \) – электрическая постоянная.

На электрической схеме конденсатор обозначается:

Виды конденсаторов:

  • по типу диэлектрика – воздушный, бумажный и т. д.;
  • по форме – плоский, цилиндрический, сферический;
  • по электроемкости – постоянной и переменной емкости.

Конденсаторы можно соединять между собой.

Параллельное соединение конденсаторов

При параллельном соединении конденсаторы соединяются одноименно заряженными обкладками. Напряжения конденсаторов равны:

Общая емкость:

Последовательное соединение конденсаторов

При последовательном соединении конденсаторов соединяют их разноименно заряженные обкладки.

Заряды конденсаторов при таком соединении равны:

Общее напряжение:

Величина, обратная общей емкости:

При таком соединении общая емкость всегда меньше емкостей отдельных конденсаторов.

Важно!
Если конденсатор подключен к источнику тока, то разность потенциалов между его обкладками не изменяется при изменении электроемкости и равна напряжению источника. Если конденсатор заряжен до некоторой разности потенциалов и отключен от источника тока, то его заряд не изменяется при изменении электроемкости.

Применение конденсаторов
Конденсаторы используются в радиоэлектронных приборах как накопители заряда, для сглаживания пульсаций в выпрямителях переменного тока.

Энергия электрического поля конденсатора

Энергия заряженного конденсатора равна работе внешних сил, которую необходимо затратить, чтобы зарядить конденсатор.

Электрическая энергия конденсатора сосредоточена в пространстве между обкладками конденсатора, то есть в электрическом поле, поэтому ее называют энергией электрического поля. Формулы для вычисления энергии электрического поля:

Так как напряженность электрического поля прямо пропорциональна напряжению, то энергия электрического поля конденсатора пропорциональна квадрату напряженности.

Плотность энергии электрического поля:

где ​\( V \)​ – объем пространства между обкладками конденсатора.

Плотность энергии не зависит от параметров конденсатора, а определяется только напряженностью электрического поля.

Основные формулы раздела «Электрическое поле»

Электрическое поле

3 (60.33%) 60 votes

15.2: Электрические поля — K12 LibreTexts

  1. Последнее обновление
  2. Сохранить как PDF
  1. Электрическое поле
  2. Сводка
  3. Обзор
  4. Узнать больше
  5. Дополнительные ресурсы
Рисунок 15.2.1

Плазменный шар, такой как тот, что изображен выше, заполнен смесью благородных газов и имеет высоковольтный электрод в центре. Линии закрутки — это линии электрического разряда, которые соединяются от внутреннего электрода к внешнему стеклянному изолятору. Когда руку кладут на поверхность земного шара, весь электрический разряд проходит прямо в эту руку.

Электрическое поле

Закон

Кулона дает нам формулу для расчета силы, действующей на один заряд другим зарядом.Однако в некоторых случаях испытательный заряд испытывает электрическую силу без видимой причины. То есть, как наблюдатели, мы не можем увидеть или обнаружить исходный заряд, создающий электрическую силу. Майкл Фарадей решил эту проблему, разработав концепцию электрического поля . Согласно Фарадею, заряд создает вокруг себя электрическое поле во всех направлениях. Если второй заряд помещается в какую-то точку поля, второй заряд взаимодействует с полем и испытывает электрическую силу.Таким образом, наблюдаемое нами взаимодействие происходит между пробным зарядом и полем, и вторая частица на некотором расстоянии больше не нужна.

Напряженность электрического поля определяется точка за точкой и может быть определена только по наличию испытательного заряда. Когда положительный тестовый заряд q t , помещается в электрическое поле, поле оказывает на заряд силу. Напряженность поля можно измерить, разделив силу на заряд испытательного заряда. Напряженность электрического поля обозначается символом E, а ее единица измерения — ньютоны / кулон.

E = F на qt / q t

Испытательный заряд можно перемещать из одного места в другое в пределах электрического поля до тех пор, пока все электрическое поле не будет отображено в единицах напряженности электрического поля .

Пример \ (\ PageIndex {1} \)

Положительный испытательный заряд 2,0 × 10–5 Кл помещают в электрическое поле. Сила, действующая на испытательный заряд, составляет 0,60 Н. Какова напряженность электрического поля в месте нахождения испытательного заряда?

Решение

E = F / q = 0.60 Н / 2,0 × 10 −5 C = 3,0 × 10 4 Н / З

Запустите симуляцию хоккея ниже и попробуйте использовать электрическое поле, чтобы помочь вам забить гол:

Сводка

  • Электрическое поле окружает каждый заряд и действует на другие заряды поблизости.
  • Напряженность электрического поля обозначается символом E и измеряется в ньютонах / кулонах.
  • Уравнение для напряженности электрического поля: E = F / q.

Обзор

  1. Вес протона 1.64 × 10 −26 Н. Заряд протона равен +1,60 × 10 −19 С. Если протон поместить в однородное электрическое поле так, что электрическая сила, действующая на протон, просто уравновесит его вес, что будет величина и направление поля?
  2. Отрицательный заряд 2,0 × 10 -8 Кл испытывает силу 0,060 Н справа в электрическом поле. Какова величина и направление поля?
  3. Положительный заряд 5,0 × 10 –4 Кл находится в электрическом поле, которое проявляет силу 2.На нем 5 × 10 −4 Н. Какова величина электрического поля в месте нахождения заряда?
  4. Если вы определили напряженность электрического поля в поле с использованием испытательного заряда 1,0 × 10 −6 C, а затем повторили процесс с испытательным зарядом 2,0 × 10 −6 C, силы, действующие на заряды, были бы равны тоже самое? Вы бы нашли значение для E?

Узнать больше

Используйте этот ресурс, чтобы ответить на следующие вопросы.

  1. Что означает, когда сила называется бесконтактной силой?
  2. Какой символ используется для обозначения напряженности электрического поля?
  3. Какая связь между направлением электрического поля и направлением электрической силы?

Дополнительные ресурсы

Учебное пособие: Учебное пособие по электростатике

Применение в реальном мире: использование статического самодельного статического электричества

Интерактивный: сенсорный экран

Видео:

Учебное пособие по физике: Напряженность электрического поля

В предыдущем разделе Урока 4 было введено понятие электрического поля.Было заявлено, что концепция электрического поля возникла в попытке объяснить силы, действующие на расстоянии. Все заряженные объекты создают электрическое поле, которое распространяется наружу в окружающее их пространство. Заряд изменяет это пространство, вызывая воздействие этого поля на любой другой заряженный объект, который входит в это пространство. Сила электрического поля зависит от того, насколько заряжен объект, создающий поле, и от расстояния до заряженного объекта. В этом разделе Урока 4 мы исследуем электрическое поле с числовой точки зрения — напряженность электрического поля .


Соотношение силы и заряда

Напряженность электрического поля — векторная величина; он имеет как величину, так и направление. Величина напряженности электрического поля определяется способом ее измерения. Предположим, что электрический заряд можно обозначить символом Q . Этот электрический заряд создает электрическое поле; Поскольку Q является источником электрического поля, мы будем называть его источником заряда .Сила электрического поля исходного заряда может быть измерена любым другим зарядом, помещенным где-то в его окрестностях. Заряд, который используется для измерения напряженности электрического поля, называется испытательным зарядом , поскольку он используется для проверки напряженности поля. Тестовый заряд имеет количество заряда, обозначенное символом q . При помещении в электрическое поле испытательный заряд испытывает электрическую силу — притягивающую или отталкивающую. Обычно эта сила обозначается символом F .Величина электрического поля просто определяется как сила, приходящаяся на заряд испытательного заряда.

Если напряженность электрического поля обозначена символом E , то уравнение можно переписать в символической форме как

.

Стандартные метрические единицы для напряженности электрического поля вытекают из его определения. Поскольку электрическое поле определяется как сила, приходящаяся на заряд, его единицами измерения будут единицы силы, разделенные на единицы заряда. В этом случае стандартными метрическими единицами измерения являются Ньютон / Кулон или Н / Кл.

В приведенном выше обсуждении вы заметите, что упоминаются два заряда — исходный заряд и тестовый заряд. Для встречи с отрядом всегда требовалось два заряда. В электрическом мире нужны двое, чтобы привлечь или оттолкнуть. Уравнение для напряженности электрического поля ( E ) имеет одну из двух величин заряда, перечисленных в нем. Поскольку задействованы два заряда, ученик должен быть предельно осторожным, чтобы использовать правильное количество заряда при вычислении напряженности электрического поля.Символ q в уравнении — это количество заряда тестового заряда (а не заряда источника). Напомним, что напряженность электрического поля определяется с точки зрения того, как она измеряется или проверяется; таким образом, тестовый заряд попадает в уравнение. Электрическое поле — это сила, приходящаяся на количество заряда на испытательном заряде .

Напряженность электрического поля не зависит от количества заряда в тестовом заряде. Если вы немного подумаете об этом заявлении, оно может вас обеспокоить.(Конечно, если вы вообще не думаете — никогда — ничто на самом деле вас не беспокоит. Невежество — это блаженство.) В конце концов, количество заряда в тестовом заряде ( q ) находится в уравнении для электрического поля. Так как же напряженность электрического поля может не зависеть от q , если q входит в уравнение? Хороший вопрос. Но если вы подумаете над этим немного дольше, вы сможете ответить на свой вопрос. (Невежество может быть блаженством. Но немного подумав, вы можете достичь прозрения, состояния, которое намного лучше, чем блаженство.) Увеличение количества заряда в тестовом заряде — скажем, в 2 раза — увеличит знаменатель уравнения в 2 раза. Но согласно закону Кулона, больший заряд также означает большую электрическую силу ( F ) . Фактически, двукратное увеличение q будет сопровождаться двукратным увеличением F . Таким образом, когда знаменатель в уравнении увеличивается в два (три или четыре) раза, числитель увеличивается во столько же раз. Эти два изменения уравновешивают друг друга, так что можно с уверенностью сказать, что напряженность электрического поля не зависит от количества заряда на тестовом заряде.Таким образом, независимо от того, какой испытательный заряд используется, напряженность электрического поля в любом заданном месте вокруг источника заряда Q будет одинакова.

Другая формула напряженности электрического поля

Вышеупомянутое обсуждение относилось к определению напряженности электрического поля с точки зрения ее измерения. Теперь мы исследуем новое уравнение, которое определяет напряженность электрического поля в терминах переменных, которые влияют на напряженность электрического поля.Для этого нам придется вернуться к уравнению закона Кулона. Закон Кулона гласит, что электрическая сила между двумя зарядами прямо пропорциональна произведению их зарядов и обратно пропорциональна квадрату расстояния между их центрами. Применительно к двум нашим зарядам — ​​исходному заряду ( Q ) и пробному заряду ( q ) — формула для электрической силы может быть записана как

Если выражение для электрической силы, заданное законом Кулона, заменить на силу в приведенном выше уравнении E = F / q, можно вывести новое уравнение, как показано ниже.

Обратите внимание, что приведенный выше вывод показывает, что тестовый сбор q был исключен как из числителя, так и из знаменателя уравнения. Новая формула для напряженности электрического поля (показанная внутри рамки) выражает напряженность поля в терминах двух переменных, которые на нее влияют. Напряженность электрического поля зависит от количества заряда на источнике заряда ( Q ) и расстояния разделения ( d ) от источника заряда.

Закон обратных квадратов

Как и все формулы в физике, формулы для напряженности электрического поля можно использовать для алгебраического решения физических задач.И, как и все формулы, эти формулы для напряженности электрического поля также могут использоваться, чтобы направлять наши размышления о том, как изменение одной переменной может (или не может) повлиять на другую переменную. Одной из особенностей этой формулы напряженности электрического поля является то, что она иллюстрирует обратную квадратичную зависимость между напряженностью электрического поля и расстоянием. Напряженность электрического поля, создаваемого зарядом источника Q , обратно пропорциональна квадрату расстояния от источника. Это известно как закон обратных квадратов .2).

Используйте этот принцип обратной квадратичной зависимости между напряженностью электрического поля и расстоянием, чтобы ответить на первые три вопроса в разделе «Проверьте свое понимание» ниже.

Повторение аналогии с вонючим полем

В предыдущем разделе Урока 4 была представлена ​​несколько грубая, но поучительная аналогия — аналогия с вонючим полем. Аналогия сравнивает понятие электрического поля, окружающего исходный заряд, с вонючим полем, окружающим вонючий подгузник младенца.Подобно тому, как каждый вонючий подгузник создает неприятное поле, каждый электрический заряд создает электрическое поле. А если вы хотите узнать силу вонючего поля, вы просто используете вонючий детектор — нос, который (насколько я знаю) всегда отталкивающе реагирует на вонючий источник. Точно так же, если вы хотите узнать силу электрического поля, вы просто используете детектор заряда — тестовый заряд, который будет реагировать притягивающим или отталкивающим образом на исходный заряд. И, конечно, сила поля пропорциональна воздействию на детектор.Более чувствительный детектор (лучший носик или более заряженный тестовый заряд) ощутит эффект более интенсивно. Тем не менее, напряженность поля определяется как влияние (или сила) на чувствительность детектора; таким образом, напряженность поля вонючего подгузника или электрического заряда не зависит от чувствительности детектора.

Если вы измерите вонючее поле подгузника, будет понятно только то, что на него не повлияет то, насколько вы вонючий. Человек, измеряющий силу вонючего поля подгузника, может создать собственное поле, сила которого зависит от того, насколько он вонючий.Но поле этого человека не следует путать с вонючим полем подгузника. Вонючее поле подгузника зависит от того, насколько вонючий подгузник. Точно так же сила электрического поля исходного заряда зависит от того, насколько заряжен исходный заряд. Кроме того, как и в случае с вонючим полем, наше уравнение электрического поля показывает, что по мере того, как вы приближаетесь к источнику поля, эффект становится все больше и больше, а напряженность электрического поля увеличивается.

Аналогия с вонючим полем оказывается полезной для передачи как концепции электрического поля, так и математики электрического поля.Концептуально он иллюстрирует, как источник поля может влиять на окружающее пространство и оказывать влияние на чувствительные детекторы в этом пространстве. И математически он показывает, как сила поля зависит от источника и расстояния от источника и не зависит от каких-либо характеристик, связанных с детектором.

Направление вектора электрического поля

Как упоминалось ранее, напряженность электрического поля является векторной величиной.В отличие от скалярной величины, векторная величина не описывается полностью, если с ней не связано направление. Величина вектора электрического поля рассчитывается как сила, приходящаяся на заряд любого заданного испытательного заряда, находящегося в пределах электрического поля. Сила на испытательном заряде могла быть направлена ​​либо на исходный заряд, либо прямо от него. Точное направление силы зависит от того, имеют ли пробный заряд и исходный заряд заряд одного и того же типа (при котором происходит отталкивание) или противоположного типа заряда (при котором происходит притяжение).Чтобы решить дилемму, направлен ли вектор электрического поля к источнику заряда или от него, было принято соглашение. Согласно всемирному соглашению, которое используется учеными, направление вектора электрического поля определяется как направление, в котором положительный тестовый заряд толкается или вытягивается в присутствии электрического поля. Используя условное обозначение положительного тестового заряда, каждый может согласовать направление E .

Учитывая это соглашение о положительном испытательном заряде, можно сделать несколько общих выводов о направлении вектора электрического поля.Положительный заряд источника создает электрическое поле, которое оказывает отталкивающее действие на положительный испытательный заряд. Таким образом, вектор электрического поля всегда будет направлен от положительно заряженных объектов. С другой стороны, положительный тестовый заряд будет притягиваться к отрицательному заряду источника. Следовательно, векторы электрического поля всегда направлены в сторону отрицательно заряженных объектов. Вы можете проверить свое понимание направлений электрического поля, ответив на вопросы 6 и 7 ниже.

Мы хотели бы предложить… Иногда просто прочитать об этом недостаточно. Вы должны взаимодействовать с ним! И это именно то, что вы делаете, когда используете один из интерактивных материалов The Physics Classroom. Мы хотели бы предложить вам совместить чтение этой страницы с использованием нашего интерактивного приложения «Положите заряд в цель» и / или интерактивного интерфейса «Электростатические пейзажи». Оба интерактивных компонента можно найти в разделе Physics Interactives на нашем веб-сайте. Оба Interactives предоставляют увлекательную среду для изучения электрических полей и действий на расстоянии.

Проверьте свое понимание

Используйте свое понимание, чтобы ответить на следующие вопросы. По завершении нажмите кнопку, чтобы просмотреть ответы.

1. Заряд Q действует как точечный заряд, создавая электрическое поле. Его сила, измеренная на расстоянии 30 см, составляет 40 Н / К. Какова величина напряженности электрического поля, которую вы ожидаете измерить на расстоянии…

а. На расстоянии 60 см?

г. 15 см?

г. На расстоянии 90 см?

г. На расстоянии 3 см?

г. На расстоянии 45 см?


2. Заряд Q действует как точечный заряд, создавая электрическое поле. Его сила, измеренная на расстоянии 30 см, составляет 40 Н / К.Какой была бы напряженность электрического поля …

а. 30 см от источника с зарядом 2Q?

г. 30 см от источника с зарядом 3Q?

г. 60 см от источника с зарядом 2Q?

г. 15 см от источника с зарядом 2Q?

e. 150 см от источника с зарядом 0.5Q?

3. Используйте свои знания о напряженности электрического поля, чтобы заполнить следующую таблицу.

4. В приведенной выше таблице найдите по крайней мере две строки, которые иллюстрируют, что напряженность вектора электрического поля равна …

а. напрямую связано с количеством заряда на исходном заряде ( Q ).

г. обратно пропорционально квадрату разделительного расстояния ( d ).

г. независимо от количества заряда в тестовом заряде ( q ).


5. Следующая единица определенно не является стандартной единицей для выражения величины напряженности электрического поля.

кг • м / с 2 / C

Однако это может быть приемлемый блок для E . Используйте анализ единиц измерения, чтобы определить, является ли вышеуказанный набор единиц приемлемой единицей измерения напряженности электрического поля.


6.Замечено, что воздушный шар A заряжен отрицательно. Воздушный шар B оказывает отталкивающее действие на воздушный шар A. Будет ли вектор электрического поля, созданный воздушным шаром B, быть направлен к B или от B? ___________ Объясните свои рассуждения.


7. Отрицательный заряд источника ( Q ) показан на диаграмме ниже. Этот исходный заряд может создавать электрическое поле. Обозначены различные места в поле.Для каждого местоположения нарисуйте вектор электрического поля в соответствующем направлении с соответствующей относительной величиной. То есть нарисуйте длину вектора E длинной, если величина велика, и короткой, где величина мала.


Напряженность электрического поля

— Calculator.org

Что такое напряженность электрического поля?

Напряженность электрического поля определяется в точке поля как равная силе, которая была бы приложена к небольшому единичному заряду (один кулон), помещенному в эту точку.Напряженность электрического поля также известна как напряженность электрического поля и является выражением напряженности электрического поля в определенном месте.

Идея электрического поля была впервые предложена Майклом Фарадеем и создается любым электрически заряженным объектом. Это поле влияет на другие заряженные объекты в непосредственной близости от первого объекта. Направление силы дает направление электрического поля. Электрическая энергия содержится в электрических полях и пропорциональна квадрату напряженности поля.Напряженность электрического поля — это векторная величина (она имеет как величину, так и направление). Движущийся электрический заряд создает магнитное поле, и на самом деле два типа поля взаимосвязаны и могут рассматриваться как одно и то же явление, рассматриваемое с разных систем отсчета.

В системе СИ единица измерения напряженности электрического поля — ньютоны на кулон (Н / Кл) или вольт на метр (В / м).

Закон

Кулона гласит, что сила, действующая между двумя зарядами, пропорциональна произведению зарядов и обратно пропорциональна квадрату расстояния между ними.Математически:

F = k q 1 q 2 / r 2

Где q 1 и q 2 — два точечных заряда, F — сила, действующая на каждый из двух точечных зарядов, r — расстояние между зарядами, а k — константа пропорциональности. Значение k зависит от среды между двумя зарядами. Если два заряда разделены свободным пространством, то k = 1 / (4 π)

Сила, испытываемая очень маленьким испытательным зарядом q, помещенным в поле E в вакууме, определяется выражением E = F / q, где F — это сила.Испытательный заряд q должен быть бесконечно малым, чтобы он не искажал измеряемое поле.

Добавьте эту страницу в закладки в своем браузере, используя Ctrl и d или используя одну из следующих служб: (открывается в новом окне)

электричества | Определение, факты и типы

Электростатика — это изучение электромагнитных явлений, возникающих при отсутствии движущихся зарядов, то есть после установления статического равновесия. Заряды быстро достигают положения равновесия, потому что электрическая сила чрезвычайно велика.Математические методы электростатики позволяют рассчитывать распределения электрического поля и электрического потенциала по известной конфигурации зарядов, проводников и изоляторов. И наоборот, имея набор проводников с известными потенциалами, можно рассчитать электрические поля в областях между проводниками и определить распределение заряда на поверхности проводников. Электрическую энергию набора зарядов в состоянии покоя можно рассматривать с точки зрения работы, необходимой для сборки зарядов; в качестве альтернативы, можно также считать, что энергия находится в электрическом поле, создаваемом этой сборкой зарядов.Наконец, энергия может храниться в конденсаторе; энергия, необходимая для зарядки такого устройства, хранится в нем как электростатическая энергия электрического поля.

Изучите, что происходит с электронами двух нейтральных объектов, тренных друг о друга в сухой среде.

Объяснение статического электричества и его проявлений в повседневной жизни.

Encyclopædia Britannica, Inc. Посмотреть все видео к этой статье

Статическое электричество — это знакомое электрическое явление, при котором заряженные частицы передаются от одного тела к другому.Например, если два предмета трутся друг о друга, особенно если они являются изоляторами, а окружающий воздух сухой, предметы приобретают одинаковые и противоположные заряды, и между ними возникает сила притяжения. Объект, теряющий электроны, становится заряженным положительно, а другой — отрицательно. Сила — это просто притяжение между зарядами противоположного знака. Свойства этой силы описаны выше; они включены в математическое соотношение, известное как закон Кулона.Электрическая сила на заряде Q 1 в этих условиях, вызванная зарядом Q 2 на расстоянии r , задается законом Кулона,

Жирные символы в уравнении обозначают вектор характер силы, а единичный вектор — это вектор, который имеет размер один и указывает от заряда Q 2 до заряда Q 1 . Константа пропорциональности k равна 10 −7 c 2 , где c — скорость света в вакууме; k имеет числовое значение 8.99 × 10 9 ньютонов на квадратный метр на квадратный кулон (Нм 2 / C 2 ). На рисунке 1 показано усилие на Q 1 , создаваемое Q 2 . Числовой пример поможет проиллюстрировать эту силу. И Q 1 и Q 2 выбраны произвольно в качестве положительных зарядов, каждый с величиной 10 −6 кулонов. Заряд Q 1 расположен в координатах x , y , z со значениями 0.03, 0, 0 соответственно, а Q 2 имеет координаты 0, 0,04, 0. Все координаты указаны в метрах. Таким образом, расстояние между Q 1 и Q 2 составляет 0,05 метра.

Получите подписку Britannica Premium и получите доступ к эксклюзивному контенту. Подпишитесь сейчас

Величина силы F на заряде Q 1 , рассчитанная с использованием уравнения (1), составляет 3,6 ньютона; его направление показано на рисунке 1.Сила на Q 2 из-за Q 1 составляет — F , что также имеет величину 3,6 ньютона; его направление, однако, противоположно направлению F . Сила F может быть выражена через ее компоненты по осям x и y , поскольку вектор силы лежит в плоскости x y . Это делается с помощью элементарной тригонометрии из геометрии рисунка 1, а результаты показаны на рисунке 2.Таким образом, в ньютонах. Закон Кулона математически описывает свойства электрической силы между зарядами в состоянии покоя. Если заряды имеют противоположные знаки, сила будет притягивающей; притяжение будет указано в уравнении (1) отрицательным коэффициентом единичного вектора r̂. Таким образом, электрическая сила на Q 1 будет иметь направление, противоположное единичному вектору , и будет направлена ​​от Q 1 к Q 2 .В декартовых координатах это привело бы к изменению знаков компонентов силы x и y в уравнении (2).

компоненты кулоновской силы

Рисунок 2: Составляющие x и y силы F на рисунке 4 (см. Текст).

Предоставлено Департаментом физики и астрономии Мичиганского государственного университета

Как можно понять эту электрическую силу на Q 1 ? По сути, сила возникает из-за наличия электрического поля в позиции Q 1 .Поле создается вторым зарядом Q 2 и имеет величину, пропорциональную размеру Q 2 . При взаимодействии с этим полем первый заряд на некотором расстоянии либо притягивается, либо отталкивается от второго заряда, в зависимости от знака первого заряда.

Сила электрического поля и закон Кулона — стенограмма видео и урока

Напряженность электрического поля

Напряженность электрического поля определяется как сила, которую испытательный заряд +1 кулон ощущает в определенном месте, измеряется в ньютонах на кулон (Н / К).Итак, вместо того, чтобы брать датчик температуры вокруг поля фермера, чтобы нарисовать карту электрического поля, вам придется взять заряд кулонов +1. Или, иначе говоря, это сила, ощущаемая на 1 кулон заряда. Например, если напряженность электрического поля составляет 3 Н / Кл, это означает, что заряд в 1 кулон будет ощущать силу в 3 ньютона, а заряд в 2 кулона будет ощущать силу в 6 ньютонов и так далее. Заряд -1 кулон будет ощущать ту же силу в противоположном направлении: помните, противоположные заряды притягиваются, а аналогичные заряды отталкиваются.

Если мы запишем эту напряженность электрического поля в виде уравнения, мы скажем, что напряженность электрического поля E равна ощущаемой силе, F , деленной на размер заряда, q .

Итак, у нас уже есть уравнение для электрической силы между двумя точечными зарядами. На другом уроке мы представили закон Кулона, который гласит, что сила F в ньютонах равна электростатической постоянной k , умноженной на размеры двух зарядов в кулонах, деленной на расстояние между ними. в метрах, в квадрате.

Если мы включим это в уравнение напряженности электрического поля и исключим одно из q s, мы обнаружим, что напряженность электрического поля в конкретном месте равна электростатической постоянной k , умноженной на размер заряд, создающий электрическое поле, деленный на расстояние, на которое вы находитесь от этого заряда, в квадрате.

Другими словами, размер электрического поля не зависит (не зависит от него) от размера тестового заряда, который вы перемещаете по полю.Размер поля зависит только от размера заряда, который создает поле.

Пример расчета

Рассмотрим пример. Представьте, что у вас есть заряд кулонов +4 в начале координат (координаты 0,0), что на самом деле довольно велико. 9 ньютонов на кулон вправо.А теперь … мы закончили.

Тот факт, что электрическое поле является вектором, может значительно усложнить эти проблемы, если вы не будете осторожны. Если вы начнете складывать несколько зарядов, вы можете рассчитать общее поле, сложив векторы электрического поля. А когда у вас больше двух зарядов, вы можете очень легко внести в него углы. Следующее, что вы знаете, вы разбиваете электрическое поле на компоненты x и x и вычисляете общее количество x и общее количество x .Принципы векторов, обсуждаемые в других уроках, применимы к любой векторной величине, включая электрическое поле. Но первый шаг — понять основы и понять, что напряженность электрического поля не зависит от размера заряда, который вы используете для ее измерения.

Резюме урока

Поле на самом деле представляет собой просто карту количества на площади пространства. Это может быть скаляр (например, температура) или вектор (например, скорость ветра). Скалярное поле просто содержит карту чисел, например, температуры в разных местах.Векторное поле содержит векторные величины — числа с направлением. Скорость ветра является векторной величиной, потому что у нее есть не только число (скорость), но и направление. Еще одно такое векторное поле — электрическое поле. Электрическое поле — это карта электрической силы в определенной области.

Напряженность электрического поля определяется как сила, которую испытывает испытательный заряд +1 кулон в определенном месте, измеряемая в ньютонах на кулон. Заряд -1 кулон будет ощущать ту же силу в противоположном направлении: помните, противоположные заряды притягиваются, а аналогичные заряды отталкиваются.Если мы запишем это в виде уравнения, мы скажем, что напряженность электрического поля, E , равна ощущаемой силе, F , деленной на размер заряда, q .

Уравнение для расчета электрического поля, создаваемого точечным зарядом

Мы можем объединить это уравнение с законом Кулона, чтобы получить уравнение для электрического поля, создаваемого точечным зарядом. 9, умноженной на размер заряда, создающего электрическое поле, разделенный на расстояние, на которое вы находитесь от этого заряда, в квадрате.

Или, другими словами, размер электрического поля не зависит (не зависит от него) от размера используемого испытательного заряда — заряда, который вы используете для измерения поля. Размер поля зависит только от размера заряда, который создает поле.

Результаты обучения

Достигайте следующих целей по мере прохождения урока:

  • Определите и опишите различные типы полей
  • Составьте уравнение напряженности электрического поля
  • Расчет силы и направления электрического поля

18.3 Электрическое поле — Физика

Задачи обучения раздела

К концу этого раздела вы сможете делать следующее:

  • Рассчитать напряженность электрического поля
  • Создание и интерпретация чертежей электрических полей

Поддержка учителей

Поддержка учителей

Цели обучения в этом разделе помогут вашим ученикам овладеть следующими стандартами:

  • (5) Студент знает природу сил в физическом мире.Ожидается, что студент:
    • (С) описать и вычислить, как величина электрической силы между двумя объектами зависит от их зарядов и расстояния между ними.

Раздел Основные термины

электрическое поле пробный заряд

Поддержка учителей

Поддержка учителей

Спросите учащихся, видели ли они фильмы, в которых используется концепция полей , как и в силовых полях .Попросите их описать, как работают такие поля. Опишите, как гравитацию можно рассматривать как поле, окружающее массу и с которым взаимодействуют другие массы. Объясните: электрические поля очень похожи на гравитационные.

Возможно, вы слышали о силовом поле в научно-фантастических фильмах, где такие поля применяют силы в определенных позициях в космосе, чтобы удержать злодея в ловушке или защитить космический корабль от вражеского огня. Концепция поля очень полезна в физике, хотя несколько отличается от того, что вы видите в фильмах.

Поле — это способ концептуализации и отображения силы, которая окружает любой объект и действует на другой объект на расстоянии без видимой физической связи. Например, гравитационное поле, окружающее Землю и все другие массы, представляет собой гравитационную силу, которая возникла бы, если бы другая масса была помещена в заданную точку внутри поля. Майкл Фарадей, английский физик XIX века, предложил концепцию электрического поля. Если вы знаете электрическое поле, вы можете легко вычислить силу (величину и направление), приложенную к любому электрическому заряду, который вы помещаете в это поле.

Электрическое поле создается электрическим зарядом и сообщает нам силу на единицу заряда во всех точках пространства вокруг распределения заряда. Распределение заряда может быть единой точкой заряда; распределение заряда, скажем, по плоской пластине; или более сложное распределение заряда. Электрическое поле распространяется в пространство вокруг распределения заряда. Теперь рассмотрите возможность размещения пробного заряда в полевых условиях. Пробный заряд — это положительный электрический заряд, заряд которого настолько мал, что он не вызывает значительного возмущения зарядов, создающих электрическое поле.Электрическое поле действует на пробный заряд в заданном направлении. Приложенная сила пропорциональна заряду испытательного заряда. Например, если мы удвоим заряд испытательного заряда, сила, приложенная к нему, удвоится. Математически, говоря, что электрическое поле — это сила на единицу заряда, записывается как

E → = F → qtestE → = F → qtest

18,15

где мы рассматриваем только электрические силы. Обратите внимание, что электрическое поле — это векторное поле, которое направлено в том же направлении, что и сила, действующая на положительный тестовый заряд.Единицы электрического поля — N / C.

Если электрическое поле создается точечным зарядом или сферой с однородным зарядом, то величина силы между этим точечным зарядом Q и пробным зарядом определяется законом Кулона

F = k | Qqtest | r2F = k | Qqtest | r2

, где используется абсолютное значение, потому что мы учитываем только величину силы. Величина электрического поля тогда равна

E = Fqtest = k | Q | r2.E = Fqtest = k | Q | r2.

18,16

Это уравнение дает величину электрического поля, создаваемого точечным зарядом Q .Расстояние r в знаменателе — это расстояние от точечного заряда Q или от центра сферического заряда до интересующей точки.

Если испытательный заряд удалить из электрического поля, электрическое поле все еще существует. Чтобы создать трехмерную карту электрического поля, представьте, что тестовый заряд размещается в разных местах поля. В каждом месте измерьте силу, действующую на заряд, и используйте векторное уравнение E → = F → / qtestE → = F → / qtest для расчета электрического поля.Нарисуйте стрелку в каждой точке, куда вы помещаете тестовый заряд, чтобы обозначить силу и направление электрического поля. Длина стрелок должна быть пропорциональна напряженности электрического поля. Если соединить эти стрелки вместе, получатся линии. На рисунке 18.17 показано изображение трехмерного электрического поля, созданного положительным зарядом.

Рис. 18.17 Трехмерное представление электрического поля, создаваемого положительным зарядом.

Поддержка учителя

Поддержка учителя

[BL] [OL] Укажите, что все силовые линии электрического поля происходят от заряда.

[AL] Обратите внимание на то, что количество линий, пересекающих воображаемую сферу, окружающую заряд, одинаково независимо от того, какой размер сферы вы выберете. Спросите, могут ли учащиеся использовать это, чтобы показать, что количество силовых линий, пересекающих поверхность на единицу площади, показывает, что напряженность электрического поля уменьшается пропорционально квадрату расстояния.

Простое рисование силовых линий электрического поля в плоскости, пересекающей заряд, дает двумерные карты электрического поля, показанные на рисунке 18.18. Слева — электрическое поле, созданное положительным зарядом, а справа — электрическое поле, созданное отрицательным зарядом.

Обратите внимание, что линии электрического поля направлены от положительного заряда в сторону отрицательного заряда. Таким образом, положительный тестовый заряд, помещенный в электрическое поле положительного заряда, будет отталкиваться. Это согласуется с законом Кулона, который гласит, что одинаковые заряды отталкивают друг друга. Если мы поместим положительный заряд в электрическое поле отрицательного заряда, положительный заряд будет притягиваться к отрицательному заряду.Обратное верно для отрицательных тестовых зарядов. Таким образом, направление силовых линий электрического поля согласуется с тем, что мы находим с помощью закона Кулона.

Уравнение E = k | Q | / r2E = k | Q | / r2 говорит, что электрическое поле становится сильнее по мере приближения к заряду, который его генерирует. Например, на расстоянии 2 см от заряда Q ( r = 2 см) электрическое поле в четыре раза сильнее, чем на расстоянии 4 см от заряда ( r = 4 см). Глядя на рисунок 18.17 и рисунок 18.18 снова, мы видим, что силовые линии электрического поля становятся более плотными по мере приближения к заряду, который его генерирует. Фактически, плотность силовых линий электрического поля пропорциональна напряженности электрического поля!

Рисунок 18.18 Силовые линии электрического поля от двух точечных зарядов. Красная точка слева несет заряд +1 нКл, а синяя точка справа несет заряд –1 нКл. Стрелки указывают направление движения положительного тестового заряда. Линии поля становятся более плотными по мере приближения к точечному заряду.

Карты электрического поля могут быть составлены для нескольких зарядов или для более сложных распределений зарядов. Электрическое поле из-за нескольких зарядов можно найти, сложив электрическое поле от каждого отдельного заряда. Поскольку эта сумма может быть только одним числом, мы знаем, что только одна линия электрического поля может проходить через любую заданную точку. Другими словами, силовые линии электрического поля не могут, , пересекать друг друга.

На рисунке 18.19 (а) показана двумерная карта электрического поля, создаваемого зарядом + q и ближайшим зарядом — q .Трехмерная версия этой карты получается вращением этой карты вокруг оси, проходящей через оба заряда. Положительный испытательный заряд, помещенный в это поле, будет испытывать силу в направлении силовых линий в его местоположении. Таким образом, он будет отталкиваться от положительного заряда и притягиваться к отрицательному. На рисунке 18.19 (b) показано электрическое поле, создаваемое двумя зарядами — q . Обратите внимание на то, как линии поля имеют тенденцию отталкиваться друг от друга и не перекрываются. Положительный тестовый заряд, помещенный в это поле, будет притягиваться к обоим зарядам.Если вы находитесь далеко от этих двух зарядов, где «далеко» означает намного больше, чем расстояние между зарядами, электрическое поле выглядит как электрическое поле от одного заряда −2 q .

Рис. 18.19 (a) Электрическое поле, создаваемое положительным точечным зарядом (слева) и отрицательным точечным зарядом той же величины (справа). (б) Электрическое поле, создаваемое двумя равными отрицательными зарядами.

Поддержка учителя

Поддержка учителя

Попросите учащихся интерпретировать карты электрического поля.Где поле наиболее сильное? Где поле самое слабое? В каком направлении поле увеличивается или уменьшается? Где поле наиболее однородное? Могут ли они проверить, что величина обвинений одинакова на данной панели? Чем поле для двух отрицательных зарядов отличается от поля для положительного и отрицательного зарядов?

Виртуальная физика

Исследование электрического поля

Это моделирование показывает электрическое поле, создаваемое зарядами, которые вы размещаете на экране.Начните с установки верхнего флажка на панели параметров справа, чтобы отобразить электрическое поле. Перетаскивайте заряды из ведер на экран, перемещайте их и наблюдайте за электрическим полем, которое они образуют. Чтобы более точно увидеть величину и направление электрического поля, перетащите датчик электрического поля или датчик E-field из нижнего ведра и перемещайте его по экрану.

Проверка захвата

На экран помещены два положительных заряда. Какое утверждение описывает электрическое поле, создаваемое зарядами?

  1. Постоянно везде.
  2. Обнуляется возле каждого заряда.
  3. Это ноль на полпути между зарядами.
  4. Это самый сильный на полпути между зарядами.

Watch Physics

Электростатика (часть 2): интерпретация электрического поля

В этом видео объясняется, как рассчитать электрическое поле точечного заряда и как интерпретировать карты электрического поля в целом. Обратите внимание, что лектор использует d для расстояния между частицами вместо r .Обратите внимание, что точечные заряды бесконечно малы, поэтому все их заряды сосредоточены в одной точке. Когда рассматриваются более крупные заряженные объекты, расстояние между ними необходимо измерять между центрами объектов.

Проверка захвата

Верно или неверно. Если точечный заряд имеет линии электрического поля, направленные внутрь, заряд должен быть положительным.

  1. правда
  2. ложь

Рабочий пример

Какая плата?

Посмотрите на рисунок электрического поля на рисунке 18.20. Какова относительная сила и знак трех зарядов?

Рис. 18.20. Карта электрического поля от трех заряженных частиц.

Стратегия

Мы знаем, что электрическое поле исходит от положительного заряда и заканчивается отрицательным зарядом. Мы также знаем, что количество силовых линий электрического поля, которые касаются заряда, пропорционально заряду. Заряд 1 имеет 12 выходящих полей. Заряд 2 имеет шесть линий поля, входящих в него. Заряд 3 имеет 12 линий поля, входящих в него.

Решение

Силовые линии электрического поля выходят из заряда 1, так что это положительный заряд. Линии электрического поля переходят в заряды 2 и 3, поэтому они являются отрицательными зарядами. Отношение зарядов q1: q2: q3 = + 12: −6: −12q1: q2: q3 = + 12: −6: −12. Таким образом, заряды 1 и 3 по величине вдвое больше заряда 2.

Обсуждение

Хотя мы не можем определить точный заряд каждой частицы, мы можем получить много информации из электрического поля о величине и знаке зарядов и о том, где сила на пробном заряде будет наибольшей (или наименьшей).

Рабочий пример

Электрическое поле от дверной ручки

Дверная ручка, которую можно принять за сферический металлический проводник, приобретает заряд статического электричества q = -1,5 нКл. Q = -1,5 нКл. Какое электрическое поле на 1,0 см перед дверной ручкой? Диаметр дверной ручки 5,0 см.

Стратегия

Поскольку дверная ручка является проводником, весь заряд распределяется по внешней поверхности металла. Кроме того, поскольку предполагается, что дверная ручка имеет идеально сферическую форму, заряд на поверхности распределен равномерно, поэтому мы можем рассматривать дверную ручку так, как если бы весь заряд находился в центре дверной ручки.Справедливость этого упрощения будет доказана в более позднем курсе физики. Теперь нарисуйте дверную ручку и определите свою систему координат. Используйте + x + x, чтобы указать направление наружу, перпендикулярное двери, с x = 0x = 0 в центре дверной ручки (как показано на рисунке ниже).

Если диаметр дверной ручки 5,0 см, ее радиус равен 2,5 см. Нам нужно знать электрическое поле на расстоянии 1,0 см от поверхности дверной ручки, что составляет расстояние r = 2,5 см + 1,0 см = 3,5 см = 2,5 см + 1,0 см = 3.5 см от центра дверной ручки. Мы можем использовать уравнение E = k | Q | r2E = k | Q | r2, чтобы найти величину электрического поля. Направление электрического поля определяется знаком заряда, который в данном случае отрицательный.

Решение

Подставив заряд Q = -1,5 нКл = -1,5 × 10-9CQ = -1,5 нКл = -1,5 × 10-9C и расстояние r = 3,5 см = 0,035mr = 3,5 см = 0,035 м в уравнение E = k | Q | r2E = k | Q | r2 дает

E = k | Q | r2 = (8,99 × 109 Н · м2 / C2) | -1,5 · 10−9C | (0,035 м) 2 = 1,1 · 104 Н / C..

Обсуждение

Это похоже на огромное электрическое поле. К счастью, электрическое поле примерно в 100 раз сильнее (3 × 106 Н / С3 × 106 Н / Ц), чтобы вызвать разрушение воздуха и провести электричество. Кроме того, вес взрослого человека составляет около 70 кг × 9,8 м / с2≈700N70кг × 9,8 м / с2≈700N, так почему же вы не чувствуете силы, действующей на протоны в руке, когда тянетесь к дверной ручке? Причина в том, что ваша рука содержит равное количество отрицательного заряда, который отталкивает отрицательный заряд дверной ручки.Из-за поляризации в вашей руке может развиться очень небольшая сила, но вы никогда этого не заметите.

Практические задачи

15.

Какова величина электрического поля на расстоянии 20 см от точечного заряда q = 33 нКл?

  1. 7,4 × 10 3 Н / З
  2. 1,48 × 10 3 Н / З
  3. 7,4 × 10 12 Н / З
  4. 0
16.

Заряд −10 нКл находится в исходной точке. В каком направлении движется электрическое поле от точки заряда на x + 10 см?

  1. Электрическое поле направлено в сторону от отрицательных зарядов.
  2. Электрическое поле указывает на отрицательные заряды.
  3. Электрическое поле указывает на положительные заряды.
  4. Электрическое поле направлено в сторону от положительных зарядов.

Проверьте свое понимание

17.

Когда линии электрического поля сближаются, что это говорит вам об электрическом поле?

  1. Электрическое поле обратно пропорционально плотности силовых линий электрического поля.
  2. Электрическое поле прямо пропорционально плотности силовых линий электрического поля.
  3. Электрическое поле не связано с плотностью силовых линий электрического поля.
  4. Электрическое поле обратно пропорционально квадратному корню из плотности силовых линий электрического поля.
18.

Если пять линий электрического поля выходят из заряда +5 нКл, сколько линий электрического поля должно выходить из заряда +20 нКл?

  1. пять линий поля
  2. 10 линий поля
  3. 15 линий поля
  4. 20 линий поля
Напряженность электрического поля

— обзор

1.

Напряженность электрического поля на поверхности проводника

Напряженность электрического поля на поверхности проводника является основным условием выбора проводника. Высокая напряженность электрического поля на поверхности проводников вызовет общую корону проводников, не только резко увеличивая потери на корону, но и приведет ко многим другим проблемам. Таким образом, конструкция линий сверхвысокого напряжения должна ограничивать напряженность электрического поля на поверхности проводников. Напряженность электрического поля на поверхности проводников регулируется отношением максимальной напряженности электрического поля на поверхности проводников к критической напряженности электрического поля проводника.Критическая напряженность электрического поля рассчитывается по формуле «клевки», которая определяется на основе данных испытаний. Максимальная напряженность электрического поля на поверхности проводников зависит от максимального рабочего напряжения, диаметра субпроводника, конфигурации пучка фазных проводов и межфазного расстояния. Для расчета доступно множество методов. Ниже приводится описание расчета напряженности электрического поля с использованием метода последовательного зеркального отображения с высокой точностью.

Напряженность электрического поля на поверхности проводников не должна превышать 80% –85% напряженности электрического поля, вызывающего общую корону, чтобы предотвратить возникновение общей короны на проводниках.Потери на коронный разряд в проводниках не должны превышать 20% потерь сопротивления линии передачи. Из результатов расчетов следует, что, за исключением отдельных шестипучковых и семисвязных проводников, отношение максимальной напряженности электрического поля на поверхности проводников к критической напряженности электрического поля превышает 0,85, отношение остальных проводников составляет менее 0,8. –0,85 и соответствует требованиям. Следовательно, напряженность электрического поля на поверхности проводника в основном не имеет управляющего воздействия.

2.

Коэффициент помех

В настоящее время доступны три основных метода оценки уровня радиопомех: (1) Метод полу теоретического анализа. В настоящее время этот метод используется нечасто; (2) метод сравнения, то есть для оценки уровня радиопомех новых линий на основе уровня существующих линий путем сравнения параметров линий; (3) Метод функции возбуждения, то есть оценка уровня радиопомех новых линий с использованием функции возбуждения, полученной от проводников, помещенных в испытательную камеру под сильным дождем.Часто используются второй и третий методы.

Метод функции возбуждения применяется к жгуту проводов и используется в этом разделе. В расчетах дана функция возбуждения в условиях сильного дождя, Γ сильный дождь , и в ней указано, что функция возбуждения (удвоение 80%) может быть получена вычитанием 10–15 дБ из Γ сильный дождь .

Уровень радиопомех в одноконтурной схеме с фазными проводниками в треугольной конфигурации ниже, чем при горизонтальном расположении фазных проводов, а уровень радиопомех в одноконтурной схеме с центральным фазным проводом, подвешенным на V- струна ниже, чем с трехфазными проводниками, подвешенными на V-образных струнах.С точки зрения конфигурации жгутов, только проводники 6 × 900 (ChuKar) в конфигурации из шести жгутов могут соответствовать стандарту 58 дБ; в других конфигурациях жгутов все проводники, кроме 7 × LGJ-500/35, могут соответствовать стандарту 58 дБ. Следовательно, в одноконтурной линии уровень радиопомех в основном не влияет на выбор проводников в конфигурациях пучков, отличных от конфигураций из шести пучков.

3.

Результаты расчета звукового шума

Слышимый шум проводников в различных конфигурациях пучков, установленных на разных типах опор, рассчитывается с использованием формулы прогнозирования звукового шума, рекомендованной Энергетическим управлением Бонневиля (BPA).Для различных типов опор, используемых в одноконтурных линиях, слышимый шум линии с фазными проводниками в треугольной конфигурации ниже, чем в горизонтальной конфигурации, а слышимый шум линии с центральной фазой на V-образной струне ниже. чем с тремя фазами на V-образных струнах. В различных конфигурациях связок слышимый шум линии с центральной фазой на V-образной струне (треугольная конфигурация) самый низкий, а слышимый шум линии с тремя фазами на V-образной струне (горизонтальная конфигурация) самый высокий.С точки зрения контроля звукового шума рекомендуется не использовать конфигурацию с тремя фазами на V-образных струнах (горизонтальная конфигурация). Исходя из критериев контроля 55 дБ (A), минимальная площадь поперечного сечения проводов в соответствии с требованиями к звуковому шуму показана в таблице 7.

0 comments on “Приведите формулу определения напряженности электрического поля: Формула напряженности электрического поля в физике

Добавить комментарий

Ваш адрес email не будет опубликован. Обязательные поля помечены *